Averages and median

This topic has expert replies
Master | Next Rank: 500 Posts
Posts: 107
Joined: Tue Oct 07, 2014 3:50 am

Averages and median

by mallika hunsur » Thu Apr 09, 2015 4:25 am
Image


Hi,

Can anyone take a look at this and explain..?

Regards,
Mallika

GMAT/MBA Expert

User avatar
GMAT Instructor
Posts: 16207
Joined: Mon Dec 08, 2008 6:26 pm
Location: Vancouver, BC
Thanked: 5254 times
Followed by:1268 members
GMAT Score:770

by Brent@GMATPrepNow » Thu Apr 09, 2015 5:13 am
Last month, 15 homes were sold in Town X. The average sale price of the homes was $150,000 and the median sale price was $130,000. Which of the following statements must be true?

I. at least one of the homes was sold for more than $165,000
II. at least one of the homes was sold for more than $13,000 but less than $150,000
III. at least one of the homes was sold for less than $130,000

A) I only
B) II only
C) III only
D) I and II only
E) I and III only

OA A
The key word in this question is MUST. As in, which of the following MUST be true. So, if it's possible that one scenario is not true, we can eliminate it.

So, let's looks at one possible scenario and see which answer choices we can eliminate.

Aside: To make things simpler, let's divide all of the prices by 1000.

First, we'll use a nice rule that says: sum of all values = (mean)(number of values)
So, the sum of all 15 prices = ($150)(15) = $2250.

If the median is $130, then the middlemost value is $130

So, one possible scenario is:
130, 130, 130, 130, 130, 130, 130, 130, 130, 130, 130, 130, 130, 130, 130, 430

Aside: To find the last value (430), I took the sum of all 15 numbers (2250) and subtracted (14)(130)

Notice that this scenario tells us that II and III need not be true (since our scenario does not conform to either one).

Since answer choices B, C, D and E all include either II or III, we can eliminate them.

This leaves us with A, which must be the correct answer.

Cheers,
Brent
Brent Hanneson - Creator of GMATPrepNow.com
Image

User avatar
GMAT Instructor
Posts: 15539
Joined: Tue May 25, 2010 12:04 pm
Location: New York, NY
Thanked: 13060 times
Followed by:1906 members
GMAT Score:790

by GMATGuruNY » Thu Apr 09, 2015 5:16 am
Last month 15 homes were sold in town X. The average sale price of the homes were $150,000 & the median sale price was $130,000. Which of the following statements MUST be true?

1) At least one of the homes was sold for more than $165,000
2) At least one of the homes was sold for more than $130,000 & less than $150,000.
3) At least one of the homes was sold for less than $130,000.

(A) I only
(B) II only
(C) III only
(D) I and II
(E) I and III
Try to prove that the answer choices DON'T have to be true.
To make the math easier, divide all of the given numbers by 1000.

The sum of the 15 prices = 15*150 = 2250.

The following solution is similar to Brent's, with one suggestion:
Before testing cases, look for a statement that seems easy to disprove.

If it's possible to have a sum of 2250 without including a price below 130, we can eliminate any answer choice that includes III.
Since the median price is 130, it's possible that 14 of the homes were sold for 130 each.
The price of the 15th home would then be 2250 - 14*130 = 430, yielding the following list of prices:

130, 130....130, median=130, 130...130, 430.

Eliminate any answer choice that includes III, since the list above does not include a price below 130.
Eliminate C and E.

Eliminate any remaining answer choice that includes II, since the list above does not include a price between 130 and 150.
Eliminate B and D.

The correct answer is A.
Private tutor exclusively for the GMAT and GRE, with over 20 years of experience.
Followed here and elsewhere by over 1900 test-takers.
I have worked with students based in the US, Australia, Taiwan, China, Tajikistan, Kuwait, Saudi Arabia -- a long list of countries.
My students have been admitted to HBS, CBS, Tuck, Yale, Stern, Fuqua -- a long list of top programs.

As a tutor, I don't simply teach you how I would approach problems.
I unlock the best way for YOU to solve problems.

For more information, please email me (Mitch Hunt) at [email protected].
Student Review #1
Student Review #2
Student Review #3

Master | Next Rank: 500 Posts
Posts: 137
Joined: Fri Nov 13, 2015 11:01 am
Thanked: 1 times
Followed by:2 members

by Amrabdelnaby » Thu Nov 19, 2015 7:02 am
Hi Brent,

Can you please explain a little further.

I am very confused with this question. also what level do you think this question is?
Brent@GMATPrepNow wrote:
Last month, 15 homes were sold in Town X. The average sale price of the homes was $150,000 and the median sale price was $130,000. Which of the following statements must be true?

I. at least one of the homes was sold for more than $165,000
II. at least one of the homes was sold for more than $13,000 but less than $150,000
III. at least one of the homes was sold for less than $130,000

A) I only
B) II only
C) III only
D) I and II only
E) I and III only

OA A
The key word in this question is MUST. As in, which of the following MUST be true. So, if it's possible that one scenario is not true, we can eliminate it.

So, let's looks at one possible scenario and see which answer choices we can eliminate.

Aside: To make things simpler, let's divide all of the prices by 1000.

First, we'll use a nice rule that says: sum of all values = (mean)(number of values)
So, the sum of all 15 prices = ($150)(15) = $2250.

If the median is $130, then the middlemost value is $130

So, one possible scenario is:
130, 130, 130, 130, 130, 130, 130, 130, 130, 130, 130, 130, 130, 130, 130, 430

Aside: To find the last value (430), I took the sum of all 15 numbers (2250) and subtracted (14)(130)

Notice that this scenario tells us that II and III need not be true (since our scenario does not conform to either one).

Since answer choices B, C, D and E all include either II or III, we can eliminate them.

This leaves us with A, which must be the correct answer.

Cheers,
Brent

GMAT/MBA Expert

User avatar
GMAT Instructor
Posts: 16207
Joined: Mon Dec 08, 2008 6:26 pm
Location: Vancouver, BC
Thanked: 5254 times
Followed by:1268 members
GMAT Score:770

by Brent@GMATPrepNow » Thu Nov 19, 2015 8:39 am
Amrabdelnaby wrote:Hi Brent,

Can you please explain a little further.

I am very confused with this question. also what level do you think this question is?
Can you tell me what part of my solution you'd like explained further?
I'd place this question in the 650 range.

Cheers,
Brent
Brent Hanneson - Creator of GMATPrepNow.com
Image

Master | Next Rank: 500 Posts
Posts: 137
Joined: Fri Nov 13, 2015 11:01 am
Thanked: 1 times
Followed by:2 members

by Amrabdelnaby » Thu Nov 19, 2015 10:00 am
I read your answer many times and i came to this conclusion:

Because statement 1 says atleast,i tried to counter it by trying to put the numbers less than 165K

in order to do so i maximized all the numbers prior to the median by making them 130 as well.

So now i have we 8 houses that cost 8(130k) = 1040k
now we have 2250k - 1040k for the remaining 6 houses. aka 1210k for the remaining 6 houses
so now if i distribute the 1210k by the remaining 6 i will get =201K and some change :)

which means that we can't have any house less than 165K

so statement 1 must be true.

Now since we can't have any house less than 165K and the rest are 130K we won't have any houses sold between 130K and 150K so statement 2 is wrong and also statement 3 is wrong coz no house can be sold for less than 130K.

is this thinking process correct?

please advise

quote="Brent@GMATPrepNow"]
Amrabdelnaby wrote:Hi Brent,

Can you please explain a little further.

I am very confused with this question. also what level do you think this question is?
Can you tell me what part of my solution you'd like explained further?
I'd place this question in the 650 range.

Cheers,
Brent[/quote]

GMAT/MBA Expert

User avatar
GMAT Instructor
Posts: 16207
Joined: Mon Dec 08, 2008 6:26 pm
Location: Vancouver, BC
Thanked: 5254 times
Followed by:1268 members
GMAT Score:770

by Brent@GMATPrepNow » Thu Nov 19, 2015 12:01 pm
Amrabdelnaby wrote: So now i have we 8 houses that cost 8(130k) = 1040k
now we have 2250k - 1040k for the remaining 6 houses. aka 1210k for the remaining 6 houses
so now if i distribute the 1210k by the remaining 6 i will get =201K and some change :)

which means that we can't have any house less than 165K
Be careful.
The remaining 6 houses must add to 1210K and 1210k/6 =201K + change
However, this does NOT mean that NONE of the remaining 6 houses are less than 165K.
We could have the following 6 prices 160K, 160K, 160K, 160K, 160K, 410K

We CAN says that AT LEAST ONE house will be greater than $165K

Cheers,
Brent
Brent Hanneson - Creator of GMATPrepNow.com
Image